I need help for this question 3 so can anyone help me please

Answers

Answer 1
Bsjsjsjssj jdjsjsjsjsks lłsskj

Related Questions

three decreased by the product of a number and two

Answers

3-2x should be the right answer

.

Use the table provided to find an exponential model ƒ(x) = ab^x, where ƒ is the population of Arizona, and the input is the number of years since 2004. Approximate to the thousandth’s place.


ƒ(x) = 5,939,292 • (1.035)x

ƒ(x) = 5,739,879 • (0.965)x

ƒ(x) = 5,739,879 • (1.035)x

ƒ(x) = 5,739,879 • (0.035)x

Answers

Answer:

this is the same quiestion i was doing if u get answer plz send me

simplify (5^0+4^-0•5)^2​

Answers

Answer:

anything raised to the power of zero= 1

(1+1/4^½)²

(1 + 1/2)²

(3/2)²

9/4

=2.25


how do you solve this

Answers

Answer:

yo what concept is this

Step-by-step explanation:

solve the following: If 7a – 4b = 3, then b =

Answers

Answer:

D (7a-3)/4

Step-by-step explanation:

7a – 4b = 3

Subtract 7a from each side

7a-7a – 4b = 3-7a

-4b = 3 -7a

Divide by -4

-4b/-4 = (3-7a)/-4

b = (7a-3)/4

Answer:

b = (7a - 3)/4

Step-by-step explanation:

7a - 4b = 3

=> -4b = 3 - 7a

=> b = (3 - 7a)/(-4)

=> b = -(3 - 7a)/4

=> b = (-3 + 7a)/4

=> b = (7a - 3)/4


Hence find the value of x that satisfies the two equations y =
5x/2-7 and y=4x/3
simultaneously.

Answers

Answer:

x=6

Step-by-step explanation:

since y=5x/2-7, and y=4x/3

         so 5x/2-7=4x/3

              5x/2-4x/3= 7

              15x-8x/6=7

               7x/6=7

               x= 7*6/7= 6

Suppose Cab Company A charges a flat fee of $4.50 plus an additional $0.25/mile. Cab Company B charges a flat fee of $1 plus an additional $0.75/mile.

First, define variables. Then write two equations: one equation will represent the cost of choosing Cab Company A, and the other equation will represent the cost of choosing Cab Company B.

Answers

Hope this helps!

Could you go to my profile and go to the social studies questions, should be for 7 points, please help with all questions! Giving brainliest!

please help i've been stuck on this for a while

Answers

Answer:

SSS Postulate

Step-by-step explanation:

The first says 2 sides are the same

The second says 2 sides are the same

The last says 2 sides are the same

Side side side

SSS Postulate

Find the value of a.
a
52

Answers

Answer:

26

because a is gone in b than

There are 135 people in a sport centre. 73 people use the gym. 59 people use the swimming pool. 31 people use the track. 19 people use the gym and the pool. 9 people use the pool and the track. 16 people use the gym and the track. 4 people use all three facilities. How many people didn't use any facilities?​

Answers

Answer:

24 people

Step-by-step explanation:

((The numbers are up there, so I am not going to define each variable.))

For starters, there is no overlap between the double facilities groups, except the triple facility users, so:

19 + 9 + 16 - 4 = 40 people

Since there is overlap between single and double groups, you will need to subtract, so:

Gym: 73 - 19 - 16 = 38

Pool: 59 - 19 - 9 = 31

Track: 31 - 9 - 16 = 6

Total for 1 facility: 38 + 31 + 6 - 4 = 71 people

((Minus 4 because the 4 triple facility users overlap the double facility users (problem is in layers: layer 1 minus layer 2, then minus layer 3).))

Next, add the totals:

71 + 40 = 111 people (using facilities)

135 - 111 = 24 people (who didn't use any)

(((I'm not 100% sure on this answer, so if someone could check my work, that would be much appreciated.)))

Câu 1: Cho tam giác MNP cân tại P thì:
A. MN = MP B. PM = NP C. PN = PN D. Không có cạnh nào bằng nhau.

Answers

Answer:

B. PM = NP

Step-by-step explanation:

2a( 3a + b)- 3a(2a+4b)

Answers

Answer:

-10ab

STEP BY STEP EXPLANATION

2a (3a + b) -3a(2a +4b)

6a² + 2ab - 6a²- 12ab

2ab-12ab 6a²-6a²

-10ab

Hope this will help you if not then sorry :)

Answer:

[tex]2a( 3a + b)- 3a(2a+4b)[/tex]

Expand by distributing terms.

[tex]6a ^{2} +2ab-3a(2a+4b)[/tex]

Expand by distributing terms.

[tex]6a 2+2ab-(6a^{2} +12ab)[/tex]

Remove parentheses

.[tex]6a ^{2} +2ab-6a^{2} -12ab[/tex]

Collect like terms.

[tex](6a^{2}-6a^{2})+(2ab-12ab)[/tex]

[tex]=-10ab[/tex]

OAmalOHopeO

Find the value of a. Round
the nearest tenth.

Answers

Answer:

side A should be about 44cm

Hello, who can help me solve this problem?Please see the picture below and take a look

Answers

9514 1404 393

Answer:

  C

Step-by-step explanation:

It's about making up a rule that gets from one figure to the next. Here's the rule I used:

The bottom left segment is rotated 1/8 turn CW from one figure to the next.

The top middle segment is rotated 1/4 turn CW from one figure to the next.

After steps, the top middle segment will be back in the same place. The lower left segment will be in the opposite corner. This corresponds to figure C.

Phythagorean theorem help plsss rnnn

Answers

Answer:

8 m

Step-by-step explanation:

Diagonal: Hypotenuse

Let the hypotenuse (diagonal) be c, Stafford street be a, and let Silvergrove Avenue be b.

Formula: a^2 + b^2 = c^2

Lets plug them in!

6^2 + b^2 = 10^2

36 + b^2 = 100

b^2= 100 - 36

b^2 = 64

64 is the exponential value of b, meaning to lower it to its original terms, we apply the opposite of squaring. Which is finding the square root.

[tex]\sqrt{64}[/tex] = 8

Therefore Silvergrove Avenue is 8 m.

Answer:

8

Step-by-step explanation:

Pythagorean Theorem: a^2 + b^2 = c^2

c (the hypotenuse, or longest leg) = 10, a or b = 6

6^2 + b^2 = 10^2

36 + b^2 = 100

b^2 = 64

b = 8

|x-2| + |x+1| - 5 = 0

Answers

Answer:

x= 3 or x= -2

I hope this helps

(x-2) +(x+1) = 5
(x-2)+ x = 5 + 1
x + x = 6 - 2
x + x = 4
x = 2

Which graph shows a system with one solution?
Graph A
Graph B
y
Graph
SVy=
315
2
5
y=2x-1
5
-5
5
-5
y
+2y = 4x – 2
O A. Graph A
B. Graph B
O C. Graph C

Answers

Believe it is graph C

For a one-way within-subjects ANOVA, the mean difference attributed to the manipulation is in the ________, and the mean difference attributed to individual differences is in the ________ of the test statistic. numerator; numerator denominator; numerator numerator; denominator denominator; denominator

Answers

Answer:

1. numerator;

2. denominator

Step-by-step explanation:

For a one-way within-subjects ANOVA, the mean difference attributed to the manipulation is in the NUMERATOR, and the mean difference attributed to individual differences is in the DENOMINATOR of the test statistic.

PLS HELP ME ON THIS QUESTION I WILL MARK YOU AS BRAINLIEST IF YOU KNOW THE ANSWER!!
The interquartile range is determined by the formula _____________.
A. Third quartile – First quartile
B. Third quartile + first quartile
C. First quartile – third quartile
D. Maximum – minimum

Answers

Answer:

A-Third quartile-First quartile

Step-by-step explanation:

The interquartile range is found by taking the third quartile minus the first quartile. It is also found by taking the right side of the box in a box and whisker plot and subtracting the left side of the box in a box and whisker plot

which of the following could be a rational number

Answers

Answer:

a number that can be expressed as an integer or the quotient of an integer divided by a nonzero integer is known as rational numbers

Step-by-step explanation:

example 3/4,1/5

The largest U.S. standard postage stamp ever issued has a width of about
3
1 inch, which was of the height of the stamp. Find the height of the stamp.
4.

Answers

Answer:

SOLUTION: " The largest US standard postage stamp ever issued has a width of about 1 inch, which was 3/4 of the height of the stamp. Write and solve an equation to find the height of the st

A new site offers a subscription that costs 28.50 for 6 months.what is unit rate price per month? show ur work

Answers

Answer:

The answer is 4.75

Step-by-step explanation:

Since six months is 28.50 then 1 month is equal to x

       28.50: 6 months

           x     : 1 month

 After this  you cross multiple so u divide by 6 both side to get 4.75

   6x/6: 28.50/6

   x=4.75

8x+6-9x=2-x-15
Help.

Answers

Step-by-step explanation:

︎︎︎︎ ︎︎︎︎ ︎︎︎︎︎ ︎︎ ︎︎︎︎❤︎❤︎❥︎ఌ︎ꨄ︎

[tex]8x + 6 - 9x = 2 - x - 15 \\ 8x - 9x + x = 2 - 15 - 6 \\ x = 2 - 15 - 6 \\ x = = 19[/tex]

8x+6-9x=2-x-158x-9x+x=2-15-6-x+x=2-210= -19

please mark this answer as brainlist

Is the following number rational or irrational?
13
12
Choose 1 answer:
А
Rational
B
Irrational

Answers

Answer: Rational

This is because the number is in the form P/Q where P and Q are integers

We have P = -13 and Q = 12.

You can think of "rational" as in "ratio" which is very closely tied to fractions. Any rational number is a fraction of two integers. The denominator can never be zero.

Answer:

Rational

Step-by-step explanation:

A rational number is any integer, fraction, terminating decimal, or repeating decimal.

Decimal form: − 1.08 3

3 repeats itself, so it's rational.

A map in a particular city shows that a city block to be a rectangle that is 264 feet wide and 900 feet long. If a person was to walk from the top left corner to the lower right corner along the diagonal of the block, they would walk approximately _____. Round your answer to the nearest whole foot. Enter only the number.

Answers

Answer:

1164

Step-by-step explanation:

The person would walk approximately 938 feet from the top left corner to the lower right corner of the city block.

What is Pythagoras's theorem?

Pythagoras's theorem states that in a right-angled triangle, the square of one side is equal to the sum of the squares of the other two sides.

As per the given figure, the length of the hypotenuse is the distance the person would walk, and the lengths of the other two sides are 264 feet and 900 feet.

Using the formula for the Pythagorean theorem, we can set up the following equation:

c² = a² + b²

where c is the length of the hypotenuse, a is the length of one of the other sides, and b is the length of the remaining side.

In this case, we can substitute the known values for a, b, and c to get:

c² = 264² + 900²

c² = 69696 + 810000

c² = 879696

c = √879696

c = 937.92110

Rounded to the nearest whole foot.

c = 938 feet

Thus, the person would walk approximately 938 feet from the top left corner to the lower right corner of the city block.

Learn more about Pythagoras's theorem here:

brainly.com/question/343682

#SPJ2

There are 35 times as many students at Wow University as teachers. When all the students and teachers are seated in the 8544 seat auditorium, 12 seats are empty. How many students attend Wow University.


A. 237

B. 249

C. 8295

D. 8124

Answers

Answer:

C. 8296

Step-by-step explanation:

Answer:

c.8295

Step-by-step explanation:

8544-12 = 8532

8532-237 =8295

The dot plot below represents this set of data:
30, 29, 30, 28, 28, 27, 22, 28, 28, 22, 27, 26, 25, 27

Answers

Answer:

True

Step-by-step explanation:

All of them match up

The midpoint of a sègment is (6,-4) and one endpoint is (13,-2). Find the coordinates of the other endpoint.

Answers

let other one be (x,y)

We know midpoint formula

[tex]\boxed{\sf (x,y)=\left(\dfrac{x_1+x_2}{2},\dfrac{y_1+y_2}{2}\right)}[/tex]

[tex]\\ \sf\longmapsto (6,-4)=\left(\dfrac{13+x}{2},\dfrac{-2+y}{2}\right)[/tex]

[tex]\\ \sf\longmapsto \dfrac{13+x}{2}=6[/tex]

[tex]\\ \sf\longmapsto 13+x=12[/tex]

[tex]\\ \sf\longmapsto x=12-13[/tex]

[tex]\\ \sf\longmapsto x=-1[/tex]

And

[tex]\\ \sf\longmapsto \dfrac{-2+y}{2}=-4[/tex]

[tex]\\ \sf\longmapsto -2+y=-8[/tex]

[tex]\\ \sf\longmapsto y=-8+2[/tex]

[tex]\\ \sf\longmapsto y=-6[/tex]

Answer:

(- 1, - 6 )

Step-by-step explanation:

Given endpoints (x₁, y₁ ) and (x₂, y₂ ) then the midpoint is

( [tex]\frac{x_{1}+x_{2} }{2}[/tex] , [tex]\frac{y_{1}+y_{2} }{2}[/tex] ) ← midpoint formula

Use this formula on the endpoints and equate to the coordinates of the midpoint.

let the other endpoint = (x, y) , then

[tex]\frac{13+x}{2}[/tex] = 6 ( multiply both sides by 2 )

13 + x = 12 ( subtract 13 from both sides )

x = - 1

[tex]\frac{-2+y}{2}[/tex] = - 4 ( multiply both sides by 2 )

- 2 + y = - 8 ( add 2 to both sides )

y = - 6

The coordinates of the other endpoint are (- 1, - 6 )

Help anyone can help me do this question,I will mark brainlest.​

Answers

Answer:

Hello,

Step-by-step explanation:

Loi d'Ohm: U=R*I

a) U=20*0.5=10 (amp)

b) I=U/R=50/0.5=100 (amp)

An arc length is a fractional part of the
circumference of a circle. The area of a
sector is a fractional part of the area of a
circle

The stained glass circle- head
the window has a 2 -inch wide
frame. The grills divide the
semicircular glass plane into
four congruent regions

Using detailed steps, describe your
solution to the problems below

Your steps should be clear enough so that
any geometry student can complete
them
A. Find the area of the blue region
B. Find the perimeter of the outer window
frame

Answers

Answer:

Each of the 4 sectors have an area:

πR²/8 - πr²/8, whereR = 28/2 - 2 = 12 inr = 6/2 = 3 in

Find the area:

A = π/8(12² - 3² ≈ 53 in²

Outer perimeter of the frame:

P = d + πd/2 =28( 1 + π/2) ≈ 72 in
Other Questions
Please it is urgent I know there are more points there are 4 questions I will give the brainiest to the right answer (Don't spam plz ) Columbus would receive all of the following rewards for his discovery except:control of all discovered landstitle of Admiral of the Ocean Seasone-tenth of all discovered riches identify the area of a regular decagon with side length 2 m rounded to the nearest tenth Give one pair of supplementary angles and one pair of vertical angles shown in the figure below.(a) Supplementary angles:(b) Vertical angles: Required information Skip to question [The following information applies to the questions displayed below.] The Tennis Times (TTT) is a publisher of magazines. Its accounting policy for subscriptions follows: Revenues Revenues from our magazine subscription services are deferred initially and later recognized as revenue as subscription services are provided. Assume TTT (a) collected $420 million in 2018 for magazines that will be distributed later in 2018 and 2019, (b) provided $204 million of services on these subscriptions in 2018, and (c) provided $216 million of services on these subscriptions in 2019. Required: Using the information given, indicate the accounts, amounts, and accounting equation effects of transactions (a), (b), and (c). (Enter any decreases to assets, liabilities, or stockholders equity with a minus sign. Enter your answers in whole dollars.) whats the answer?? please help ANSWER PLS!! :DDWhich of the following is not a property of a regular pyramid?A. lateral faces that are parallelB. lateral faces that are congruent isosceles trianglesC. lateral edges that are congruentD. volume of the pyramid is equal to one-third the product of the area of its base and its altitude The needle in the following diagram rotates around a fixed point in the middle. Based on the information given, what way will the needle rotate when the current is applied? who is current prime minister of nepal ?? A car is travelling in the velocity of 20 m/ s . After 30 sec it covers a distance of 1200 m. Calculate the acceleration of car and the final velocity of the car Help!!! please!!! Confused! What effect will replacing x with (x4) have on the graph of the equation y=(x3)2 y = ( x 3 ) 2 ? 48-15{3(-2-4+11)}+12 If fx) = 22 - 5 and g(x) = 3+3, find (f-g)(x).. -? - 8. 3 - 2x2 - 2. 2 - 3x-2OD. 22 3 8 write each of the following fraction as equivalent fractions with a denominator of 10 a. 1/2 b. 1/4 c. 3/30 d. 12/30 Which fraction is greater than the fraction represented by the model?HURRY PLS IM BEING TIMED!!!! In your words Explain the first amendment and the right to speedy trail amendment? The surface area of a cylinder is given by the formula S = 30r + 2r2How many times greater is this compared to a circle with A = r2 1. You are given the 3rd and 5th term of an arithmetic sequence. Describe in words how to determine the general term. 2. You are given the 3rd and 5th term of an geometric sequence. Describe how to determine the 10th term without finding the general term. .Prove a+ar+ar2++arn-1=a(rn -1 ) r-1